LSAT and Law School Admissions Forum

Get expert LSAT preparation and law school admissions advice from PowerScore Test Preparation.

User avatar
 Dave Killoran
PowerScore Staff
  • PowerScore Staff
  • Posts: 5852
  • Joined: Mar 25, 2011
|
#44156
Complete Question Explanation
(The complete setup for this game can be found here: lsat/viewtopic.php?t=13729)

The correct answer choice is (D)

The condition in the question stem yields the following scenario (Not Laws from the original diagram are included):
J03_Game_#3_#15_diagram 1.png
At this point, we can see that Honolulu has become quite restricted, and therefore H and V must be connected, with the further implication that because V is connected to H, then V must also be connected to T:
J03_Game_#3_#15_diagram 2.png
From this information, we can eliminate answer choices (A), (B), (D), and (E). Therefore, answer choice (D) is correct.

Note that answer choice (E) is impossible because of the action of the second rule.
You do not have the required permissions to view the files attached to this post.

Get the most out of your LSAT Prep Plus subscription.

Analyze and track your performance with our Testing and Analytics Package.